Đến nội dung

Hình ảnh

[CHUYÊN ĐỀ] CHỨNG MINH BẤT ĐẲNG THỨC


  • Please log in to reply
Chủ đề này có 166 trả lời

#61
Chung Anh

Chung Anh

    Sĩ quan

  • Thành viên
  • 420 Bài viết

48. Cho a; b; c là các số thực không âm thỏa mãn điều kiện ab + bc + ca = 1. Chứng minh:

                        $\frac{1}{a+b}+\frac{1}{b+c}+\frac{1}{c+a}\geq \frac{5}{2}$

 

P/s: Mọi người còn bài CM bđt nào thì post lên để cùng thảo luận nhé!

Áp dụng bất đẳng thức Iran 96 có

$(ab+bc+ca)\left ( \frac{1}{(a+b)^2}+\frac{1}{(b+c)^2}+\frac{1}{(a+c)^2} \right )\geq \frac{9}{4}=>\left (  \frac{1}{(a+b)^2}+\frac{1}{(b+c)^2} +\frac{1}{(a+c)^2}\right )\geq \frac{9}{4} $

Lại có $2\sum \frac{1}{(a+b)(b+c)}=\frac{4(a+b+c)}{(a+b)(b+c)(c+a)}=\frac{4(ab+bc+ca)}{(a+b)(b+c)(c+a)}=4+\frac{4abc}{(a+b)(b+c)(c+a)}\geq 4 $

=>$=>VT\geq \sqrt{\frac{9}{4}+4}=\frac{5}{2} $

Dấu bằng xảy ra $<=>(a,b,c)=(1,1,0)$ và các hoán vị


Chung Anh


#62
Chung Anh

Chung Anh

    Sĩ quan

  • Thành viên
  • 420 Bài viết

 

59. Cho a; b; c là các số thực dương. Chứng minh rằng:

                        $\frac{a^{3}}{b^{2}}+\frac{b^{3}}{c^{2}}+\frac{c^{3}}{a^{2}}\geq \frac{a^{2}}{b}+\frac{b^{2}}{c}+\frac{c^{2}}{a}$

 

 

P/s: Mọi người còn bài CM bđt nào thì post lên để cùng thảo luận nhé!

Có $a^3+b^3 \geq a^2b+b^2a <=> (a-b)^2(a+b) \geq 0$

=>$\frac{a^3+b^3}{b^2}\geq \frac{a^2b+b^2a}{b^2}\Leftrightarrow \frac{a^3}{b^2}+b\geq \frac{a^2}{b}+a $

Thiết lập các BĐT tương tự cộng lại có đpcm


Bài viết đã được chỉnh sửa nội dung bởi Chung Anh: 06-08-2015 - 21:01

Chung Anh


#63
nloan2k1

nloan2k1

    Thượng sĩ

  • Thành viên
  • 219 Bài viết

$a^3+b^2 \geq a^2b+b^2a <=> (a-b)^2(a+b) \geq 0$

=>$\frac{a^3+b^3}{b^2}\geq \frac{a^2b+b^2a}{b^2}\Leftrightarrow \frac{a^3}{b^2}+b\geq \frac{a^2}{b}+a $

Thiết lập các BĐT tương tự cộng lại có đpcm

Hai $BĐT$ này đâu có tương đương nhau đâu chị? 



#64
HoangVienDuy

HoangVienDuy

    Sĩ quan

  • Thành viên
  • 309 Bài viết
 

53. Cho a; b; c là các số thực dương. Chứng minh rằng:

                        $\frac{a}{(b+c)^{2}}+\frac{b}{(c+a)^{2}}+\frac{c}{(a+b)^{2}}\geq \frac{9}{4(a+b+c)}$

P/s: Mọi người còn bài CM bđt nào thì post lên để cùng thảo luận nhé!

Áp dụng Bunyacowski ta có:

$(a+b+c)\left ( \sum \frac{a}{(b+c)^{2}} \right )\geq \left (\sum \frac{a}{b+c} \right )^{2}$

ta sẽ chứng minh $\left [ \sum \frac{a}{b+c} \right ]^{2}\geq \frac{9}{4}\Leftrightarrow \sum \frac{a}{b+c}\geq \frac{3}{2}$ (luôn đúng vì đây là bdt nesbit)


Bài viết đã được chỉnh sửa nội dung bởi HoangVienDuy: 06-08-2015 - 20:36

Có một người đi qua hoa cúc

Có hai người đi qua hoa cúc

Bỏ lại sau lưng cả tuổi thơ mình...

FB:https://www.facebook.com/hoang.vienduy


#65
congdan9aqxk

congdan9aqxk

    Thượng sĩ

  • Thành viên
  • 215 Bài viết

 

56. Cho a; b; c là các số thự dương thỏa mãn điều kiện a + b + c = 1. Chứng minh rằng:

                        $5(a^{2}+b^{2}+c^{2})\leq 6(a^{3}+b^{3}+c^{3})+1$ (1)

 

P/s: Mọi người còn bài CM bđt nào thì post lên để cùng thảo luận nhé!

(1)$\Leftrightarrow 5(\sum a^{2})(\sum a)\leq 6(\sum a^{3})+(a+b+c)^{3}$ (vì a+b+c=1)

Nhân ra rồi thu gọn ta đươc:$\sum ab(a+b)\leq \sum a^{3}+3abc$ (đây là bđt Schur bậc 3)



#66
congdan9aqxk

congdan9aqxk

    Thượng sĩ

  • Thành viên
  • 215 Bài viết

 

49. Cho a; b; c là các số thực dương. Chứng minh rằng:

                        $\sqrt{a^{2}+(1-b)^{2}}+\sqrt{b^{2}+(1-c)^{2}}+\sqrt{c^{2}+(1-a)^{2}}\geq \frac{3\sqrt{2}}{2}$

50. Cho $a;b;c\in (0;1)$. Chứng minh rằng:

                        $\sqrt{abc}+\sqrt{(1-a)(1-b)(1-c)}<1$

 

P/s: Mọi người còn bài CM bđt nào thì post lên để cùng thảo luận nhé!

Bài 49 dùng mincopski,được VT$\geq \sqrt{t^{2}+(3-t)^{2}}$ (t=a+b+c)

VT$\geq VP\Leftrightarrow 2t^{2}-6t+9\geq \frac{9}{2}\Leftrightarrow (2t-3)^{2}\geq 0$ (đúng)

Bài 50

Ta có  $0\leq abc\leq 1$ và 1/2>1/3 nên $\sqrt{abc}\leq \sqrt[3]{abc}\leq \frac{a+b+c}{3}$

Tương tự có:$\sqrt{(1-a)(1-b)(1-c)}\leq \frac{3-a-b-c}{3}$

Cộng vế theo vế 2 bđt ta được VT<=1

dấu = xảy ra khi trong 3 số a;b;c có ít nhất 1 số =1(không thỏa mãn đk)

Vậy $\sqrt{abc}+\sqrt{(1-a)(1-b)(1-c)}<1$



#67
congdan9aqxk

congdan9aqxk

    Thượng sĩ

  • Thành viên
  • 215 Bài viết

 

54. Cho a; b; c $\geq$ 0. Chứng minh rằng: 

$\sqrt{a^{4}+a^{2}b^{2}+b^{4}}+\sqrt{b^{4}+b^{2}c^{2}+c^{4}}+\sqrt{c^{4}+c^{2}a^{2}+a^{4}}\geq a\sqrt{2a^{2}+bc}+b\sqrt{2b^{2}+ca}+c\sqrt{2c^{2}+ab}$

 

 

P/s: Mọi người còn bài CM bđt nào thì post lên để cùng thảo luận nhé!

Ta có $(a^{2}-b^{2})^{2}\geq 0\Rightarrow \sqrt{ a^{4}+a^{2}b^{2}+b^{4}}\geq \sqrt{3}\frac{a^{2}+b^{2}}{2}\Rightarrow VT\geq \sqrt{3}(\sum a^{2})\geq \frac{3}{4}(a^{2}+b^{2})^{2}$

Theo cauchy-schawrs,có

VP$\leq \sqrt{(\sum a^{2})(2a^{2}+2b^{2}+2c^{2}+ab+ac+bc)}\leq \sqrt{3}(\sum a^{2})$ (Vì $ab+ac+bc\leq \sum a^{2}$ )


Bài viết đã được chỉnh sửa nội dung bởi congdan9aqxk: 06-08-2015 - 21:35


#68
lethutang7dltt

lethutang7dltt

    Trung sĩ

  • Thành viên
  • 117 Bài viết

Bài 22: Cho các số dương a, b, c, d. Biết $\frac{a}{1+a}+\frac{b}{1+b}+\frac{c}{1+c}+\frac{d}{1+d}\leq 1$

 

22.$\frac{b}{b+1}+\frac{c}{c+1}+\frac{d}{d+1}\leq 1-\frac{a}{a+1}=\frac{1}{a+1} \Rightarrow \frac{1}{a+1}\geq 3\sqrt[3]{\frac{bcd}{(b+1)(c+1)(d+1)}}$

cmtt rồi nhân 4 BĐT  theo vế $=>1\geq 81abcd=>đpcm$


#oimeoi  :wub: #


#69
hoduchieu01

hoduchieu01

    Thượng sĩ

  • Thành viên
  • 208 Bài viết

61 . $\frac{2 (a^{3}+b^{3}+c^{3})}{abc}$ +  $\frac{9(a+b+c)^{2}}{a^{2}+b^{2}+c^{2}}$  lớn hơn hoặc bằng    33


Bài viết đã được chỉnh sửa nội dung bởi hoduchieu01: 11-08-2015 - 23:25


#70
hoduchieu01

hoduchieu01

    Thượng sĩ

  • Thành viên
  • 208 Bài viết

bài 61 này gợi ý làm theo phương pháp S.O.S nhé


Bài viết đã được chỉnh sửa nội dung bởi hoduchieu01: 11-08-2015 - 23:25


#71
Dinh Xuan Hung

Dinh Xuan Hung

    Thành viên nổi bật 2015

  • Thành viên nổi bật 2016
  • 1396 Bài viết

 

51. Cho a; b; c là các số thực dương thỏa mãn điều kiện abc = 1. Chứng minh rằng:

                        $\frac{b+c}{\sqrt{a}}+\frac{c+a}{\sqrt{b}}+\frac{a+b}{\sqrt{c}}\geq \sqrt{a}+\sqrt{b}+\sqrt{c}+3$

 

 

P/s: Mọi người còn bài CM bđt nào thì post lên để cùng thảo luận nhé!

$\frac{b+c}{\sqrt{a}}+\frac{c+a}{\sqrt{b}}+\frac{a+b}{\sqrt{c}}=\frac{b}{\sqrt{a}}+\frac{c}{\sqrt{b}}+\frac{a}{\sqrt{c}}+\frac{c}{\sqrt{a}}+\frac{a}{\sqrt{b}}+\frac{b}{\sqrt{c}}\geq \frac{(\sum \sqrt{a})^2}{\sum \sqrt{a}}+\frac{(\sum \sqrt{a})^2}{\sum \sqrt{a}}=\sum \sqrt{a}+\left ( \sqrt{a}+\sqrt{b}+\sqrt{c} \right )\geq \sum \sqrt{a}+3\sqrt[3]{\sqrt{(abc)}}=\sum \sqrt{a}+3$



#72
Boruto

Boruto

    Binh nhất

  • Thành viên
  • 33 Bài viết

cách khác cho câu 59 

ta có $\frac{a^3}{b^2}+a\geq \frac{2a^2}{b}$  do đó $\sum \frac{a^3}{b^2}\geq \sum \frac{2a^2}{b}-(a+b+c)$

cần cm $a+b+c\leq \frac{a^2}{b}+\frac{b^2}{c}+\frac{c^2}{a}$  (ĐÚNG THEO CAUCHY SCHWAR)

 

do do >= dpcm



#73
Boruto

Boruto

    Binh nhất

  • Thành viên
  • 33 Bài viết

câu 57  :icon6:  :icon6:

Ta có $abc\leq 1 \rightarrow abc\leq \frac{(a+b+c)^3}{27}\leq 1 \rightarrow a+b+c\leq 3$

 

BDT: $\frac{a}{b}+\frac{b}{c}+\frac{c}{a}=\frac{a^2}{ab}+\frac{b^2}{bc}+\frac{c^2}{ca}\geq \frac{(a+b+c)^2}{ab+bc+ca}$

 

mat khac $ab+bc+ca\leq \frac{(a+b+c)^2}{3}\leq a+b+c$  (DO $ab+bc+ca\leq \frac{(a+b+c)^2}{3}= \frac{(a+b+c)(a+b+c)}{3}\leq \frac{3(a+b+c)}{3}=a+b+c$

 

do đó $\sum \frac{a}{b}\geq \frac{(a+b+c)^2}{a+b+c}= a+b+c$  :closedeyes:


Bài viết đã được chỉnh sửa nội dung bởi Boruto: 15-08-2015 - 15:02


#74
cachcach10x

cachcach10x

    Hạ sĩ

  • Thành viên
  • 76 Bài viết

62.Phương trình bậc hai $x^{2}+mx+n=0 (n\leq m-1)$ có hai nghiệm là a và b. CMR:$a^{2}+b^{2}\geq 1$

P/s: Bài này chẳng biết là BĐT hay phương trình nữa (thấy nó trong tập BĐT của chị mình nên cho vào BĐT) :wacko:  :wacko: .Bài này trích Đề thi chọn HSG toán 9 toàn quốc bảng B, 1994- 1995.


A naughty girl :luoi:  :luoi:  :luoi:  


#75
Hoang Nhat Tuan

Hoang Nhat Tuan

    Hỏa Long

  • Thành viên
  • 974 Bài viết

62.Phương trình bậc hai $x^{2}+mx+n=0 (n\leq m-1)$ có hai nghiệm là a và b. CMR:$a^{2}+b^{2}\geq 1$

P/s: Bài này chẳng biết là BĐT hay phương trình nữa (thấy nó trong tập BĐT của chị mình nên cho vào BĐT) :wacko:  :wacko: .Bài này trích Đề thi chọn HSG toán 9 toàn quốc bảng B, 1994- 1995.

Ta có: $\Delta =m^2-4n\geq 0=>m^2\geq 4n$

Sử dụng Vi-et thì: $a+b=-m;ab=n$

Khi đó: $a^2+b^2=m^2-2n\geq m^2-2(m-1)=m^2-2m+2\geq 1<=>(m-1)^2\geq 0$

BĐT được chứng minh

Spoiler


Ngài có thể trói cơ thể tôi, buộc tay tôi, điều khiển hành động của tôi: ngài mạnh nhất, và xã hội cho ngài thêm quyền lực; nhưng với ý chí của tôi, thưa ngài, ngài không thể làm gì được.

#76
cachcach10x

cachcach10x

    Hạ sĩ

  • Thành viên
  • 76 Bài viết

Bài 62 còn cách giải nào khác không ạ?


Bài viết đã được chỉnh sửa nội dung bởi cachcach10x: 16-08-2015 - 09:01

A naughty girl :luoi:  :luoi:  :luoi:  


#77
Namthemaster1234

Namthemaster1234

    Thiếu úy

  • Thành viên
  • 550 Bài viết

61 . $\frac{2 (a^{3}+b^{3}+c^{3})}{abc}$ +  $\frac{9(a+b+c)^{2}}{a^{2}+b^{2}+c^{2}}$  lớn hơn hoặc bằng    33

Chú ý rằng: $\frac{9(a+b+c)^{2}}{a^{2}+b^{2}+c^{2}} \geq \frac{81abc}{a^3+b^3+c^3}$


Bài viết đã được chỉnh sửa nội dung bởi Namthemaster1234: 01-09-2015 - 17:17

Đừng lo lắng về khó khăn của bạn trong toán học, tôi đảm bảo với bạn rằng những khó khăn toán học của tôi còn gấp bội.
(Albert Einstein)

Visit my facebook: https://www.facebook.com/cao.simon.56

:icon6: :icon6: :icon6: :icon6: :icon6:


#78
hoduchieu01

hoduchieu01

    Thượng sĩ

  • Thành viên
  • 208 Bài viết

Chú ý rằng: $\frac{9(a+b+c)^{2}}{a^{2}+b^{2}+c^{2}} \geq \frac{81abc}{a^3+b^3+c^3}$

Bất đẳng thức j vậy bạn mình chưa biết?

 

59
nhân a+b+c dùng cauchy scwarz inequality bài này là JBMO 2002 shortlist

 

57 trích trong gazeta matematica
Dùng bddt cô si



#79
CaoHoangAnh

CaoHoangAnh

    Trung sĩ

  • Thành viên
  • 107 Bài viết

Áp dụng bất đẳng thức Iran 96 có

$(ab+bc+ca)\left ( \frac{1}{(a+b)^2}+\frac{1}{(b+c)^2}+\frac{1}{(a+c)^2} \right )\geq \frac{9}{4}=>\left (  \frac{1}{(a+b)^2}+\frac{1}{(b+c)^2} +\frac{1}{(a+c)^2}\right )\geq \frac{9}{4} $

Lại có $2\sum \frac{1}{(a+b)(b+c)}=\frac{4(a+b+c)}{(a+b)(b+c)(c+a)}=\frac{4(ab+bc+ca)}{(a+b)(b+c)(c+a)}=4+\frac{4abc}{(a+b)(b+c)(c+a)}\geq 4 $

=>$=>VT\geq \sqrt{\frac{9}{4}+4}=\frac{5}{2} $

Dấu bằng xảy ra $<=>(a,b,c)=(1,1,0)$ và các hoán vị

Cho mình công thức tổng quát của bất đẳng thức Iran 96 đi



#80
the unknown

the unknown

    Thượng sĩ

  • Thành viên
  • 208 Bài viết

Cho $x,y,z$ là các số thực dương thỏa $x+y+z=3$. Chứng minh rằng:

                        $\sqrt{x}+\sqrt{y}+\sqrt{z}\geq xy+yz+zx$


$\texttt{If you don't know where you are going, any road will get you there}$





0 người đang xem chủ đề

0 thành viên, 0 khách, 0 thành viên ẩn danh